Inscription / Connexion Nouveau Sujet
Niveau exercices
Partager :

suite et limite

Posté par
carpediem
21-12-19 à 15:08

salut

c'est les vacances alors un petit pb ... dont je n'ai pas la réponse ...

pour n entier non nul on considère la fonction f_n  :  x \mapsto x - n - \dfrac {\ln x} x et les suites (u_n) et (v_n) dont les termes sont les solutions de l'équation f_n(x) = 0 avec u_n \le v_n

u_n  et  v_n   tendent respectivement vers 0 et +oo mais ...

quelle est la limite de la suite (w_n) avec w_n = u_n v_n ? si tant est qu'elle en a une ...

éventuellement peut-on en avoir un développement asymptotique ?

have some fun

PS : il n'est pas nécessaire de blanker ... plutôt un travail collaboratif tendant vers une plus juste vérité ...

Posté par
lake
re : suite et limite 21-12-19 à 16:38

Bonjour,

Que des conjectures pour l'instant:

  v_n\geq n (ça, c'est facile à prouver).

  u_n\geq \dfrac{\ln\,n}{2n} (ça, c'est une conjecture dont je ne suis pas absolument sûr).

Il faudrait donc prouver que f_n\left(\dfrac{\ln\,n}{2n}\right)>0
 \\

Posté par
carpediem
re : suite et limite 21-12-19 à 17:04

pour v_n on peut faire mieux :

posons v_n = n + r_n

f_n(v_n) = 0 \iff r_n - \dfrac {\ln n + \ln (1 + \dfrac {r_n} n)} {n \left( 1 + \dfrac {r_n} n \right)} = 0 \iff r_n = \dfrac 1 n \left(\ln n + \dfrac {r_n} n + o(1/n) \right) \left( 1 - \dfrac {r_n} n + o(1/n) \right)

donc r_n = \dfrac {\ln n} n + o(1/n) ce me semble-t-il ...

mais pour u_n je n'ai aucune idée ...

Posté par
lake
re : suite et limite 21-12-19 à 17:20

Oui, ça à l'air vrai mais il faudrait préciser que:

  0<r_n<1  (puisque f_n(n+1)>0) et que le petit o final est positif non ?

Posté par
lake
re : suite et limite 21-12-19 à 17:27

Finalement inutile pour le petit o si on procède par équivalents.

Posté par
carpediem
re : suite et limite 21-12-19 à 18:25

ben disons qu'on a n \le v_n \le n + 1 => 0 \le r_n \le 1 (et on peut même mettre des inégalités strictes ...

je ne pense pas que le signe de o(1/n) importe ...

par contre s'arrêter à l'ordre 1 est peut-être insuffisant suivant ce que donne u_n ... dont je n'ai aucune idée ...

tu donnes une minoration de u_n (conjecture) mais il nous faudrait en fait un encadrement (pour tenter le théorème de gendarmes éventuellement par la suite)

pour v_n en fait il faudrait poser v_n = n + \dfrac {\ln n} n + r_n ... et recommencer !!!

Posté par
ty59847
re : suite et limite 21-12-19 à 19:15

En faisant quelques expériences, on constate que un vn est très proche de -ln(un).
Reste à le démontrer.

Posté par
ty59847
re : suite et limite 21-12-19 à 19:16

u_n v_n très proche de ln(u_n)

Posté par
LittleFox
re : suite et limite 21-12-19 à 22:18

En fait v_n tend vers n et u_n tend vers \frac{W(n)}{n} quand n tend vers l'infini.

Leur produit tend donc vers  W(n).

Or W(n) a pour développement asymptotique  en plus l'infini ln(n) - ln(ln(n)) + \frac{ln(ln(n))}{ln(n)} + O((\frac{ln(ln(n))}{ln(n)})^2)

Avec W(n) la fonction W de Lambert

C'est pas très clair comment je suis arrivé à cette conclusion, j'ai fait beaucoup d'approximation et quelques erreurs Mais ça marche bien .

Posté par
lafol Moderateur
re : suite et limite 21-12-19 à 22:26

bonjour

LittleFox @ 21-12-2019 à 22:18

En fait v_n tend vers n et u_n tend vers \frac{W(n)}{n} quand n tend vers l'infini.

Leur produit tend donc vers W(n).
.


Posté par
lake
re : suite et limite 22-12-19 à 09:21

Citation :
tu donnes une minoration de u_n (conjecture) mais il nous faudrait en fait un encadrement (pour tenter le théorème de gendarmes éventuellement par la suite)


Si la conjecture u_n\geq \dfrac{\ln\,n}{2n} est correcte, avec v_n\geq n, on a:

   u_n.v_n\geq \dfrac{1}{2}\,\ln\,n

Ce qui permet de conclure.

Posté par
lake
re : suite et limite 22-12-19 à 10:10

Citation :
Si la conjecture u_n\geq \dfrac{\ln\,n}{2n} est correcte...


Je suis maintenant persuadé qu'elle l'est.

Il faut donc prouver que f_n\left(\dfrac{\ln\,n}{2n}\right)>0 pour tout n>1.

Posté par
LittleFox
re : suite et limite 22-12-19 à 10:38

@lafol

Je sens que j'ai fait quelque chose d'horrible pour un mathématicien mais étant un ingénieur, je ne vois pas

Peux tu être plus explicite?

Posté par
lake
re : suite et limite 22-12-19 à 10:40

Soit n>1.

Après quelques (petits) calculs:

f_n\left(\dfrac{\ln\,n}{2n}\right)=\dfrac{\ln\,n}{2n}+\dfrac{2n\,\ln\,2}{\ln\,n}+n\left(1-\dfrac{2\,\ln\,(\ln\,n)}{\ln\,n}\right)
 \\

Une étude de la fonction x\mapsto 1-\dfrac{2\,\ln\,(\ln\,x)}{\ln\,x} sur ]1,+\infty[ (avec un minimum positif en e^e) permet de prouver que la parenthèse est positive.

Donc f_n\left(\dfrac{\ln\,n}{2n}\right)\geq 0, la conjecture est prouvée et la suite (u_n.v_n) tend vers +\infty
  

Posté par
carpediem
re : suite et limite 22-12-19 à 11:01

ha oui merci lake j'ai zappé l'idée que la limite pouvait être infinie !!!

f_n(\dfrac {\ln n} {2n}) ) = \dfrac {\ln n} {2n} - n - \dfrac {2n} {\ln n} [\ln \ln n - \ln (2n)]

le numérateur est :

\ln^2 n - 2n^2 \ln n -4n^2 \ln \ln n + 2n \ln (2n) = \ln^2 n + \ln \dfrac {(2n)^{2n}} {n^{2n^2} (\ln n)^{4n^2}} = \ln^2 n + \ln \dfrac {4^n} {n^{2n^2- 2n} (\ln n)^{4n^2}}

ouais ... comment montrer que ce nombre est positif ...

Posté par
lake
re : suite et limite 22-12-19 à 11:03

Bonjour carpediem,

C'est fait au dessus

Posté par
LittleFox
re : suite et limite 22-12-19 à 11:08


f_n(n) = n - n - \frac{ln(n)}{n} = -\frac{ln(n)}{n}

\lim_{n \rightarrow \infty} f_n(n) = 0^-

f_n(\frac{W(n)}{n}) = \frac{W(n)}{n} - n - \frac{ln(\frac{W(n)}{n})}{\frac{W(n)}{n}} = \frac{W(n)}{n} - n - n\frac{ln(W(n))-ln(n)}{W(n)} = \frac{W(n)}{n} - n - n\frac{ln(n)- W(n) -ln(n)}{W(n)} \
 \\ =\frac{W(n)}{n} - n + n = \frac{W(n)}{n}

\lim_{n \rightarrow \infty} f_n(\frac{W(n)}{n}) = 0^+

C'est suffisant pour montrer que les deux racines tendent vers (se comportent comme) ln(n) et \frac{W(n)}{n}?

Posté par
LittleFox
re : suite et limite 22-12-19 à 11:10


LittleFox @ 22-12-2019 à 11:08


[...]

C'est suffisant pour montrer que les deux racines tendent vers (se comportent comme) ln(n) et \frac{W(n)}{n}?


Erratum :
C'est suffisant pour montrer que les deux racines tendent vers (se comportent comme) n et \frac{W(n)}{n}?

Posté par
carpediem
re : suite et limite 22-12-19 à 11:22

merci lake ... tout simplement !!!

je me suis bien compliqué la vie ... en voulant tout mettre (ou presque) dans un seul ln

(je n'ai pas vu ton msg au moment où j'ai commencé à bricoler)

Posté par
lake
re : suite et limite 22-12-19 à 11:28

Tout ça est quand même un tantinet laborieux.

Un perroquet ou un jandri, s'ils passaient par ici, nous sortiraient directement un développement asymptotique...

Posté par
carpediem
re : suite et limite 22-12-19 à 11:34

je ne sais pas mais ça serait avec plaisir ...

en particulier pour confirmer mon développement de 17h04 (et obtenir plus) et pour obtenir un développement de u_n aussi ...

Posté par
Ulmiere
re : suite et limite 22-12-19 à 14:50

u_n = n + \frac{\ln u_n}{u_n} \implies n-u_n = -\frac{\ln u_n}{u_n}
Puisque u_n \to 0, n-u_n\to\infty. Le développement asymptotique de la branche >-1/e de W en +infini donne

-\ln u_n = \ln (n-u_n) - \ln\ln(n-u_n) + \frac{\ln\ln n-u_n}{\ln n-u_n} + O\left(\left(\frac{\ln\ln n-u_n}{\ln n-u_n}\right)^2\right)

Je ne suis pas d'accord, sauf erreur, avec le DA de v_n plus haut car si v_n-n \to 0 on aurait
\ln v_n = -W(n-v_n) = v_n-n + o(v_n-n), ce qui impliquerait v_n \sim 1.
J'ai pu me tromper, j'ai fait les calculs vite fait, à vérifier...


L'étape suivante est de calculer un DA de n-u_n, de le factoriser par n pour avoir un DL du type ln(1+x). Ensuite, remplacer dans la formule du DA de -ln(un) ci-dessus, et (peut-être) voir apparaitre quelque chose de plus simple que cette horreur. J'ai pas le courage d'essayer

Posté par
jandri Correcteur
re : suite et limite 22-12-19 à 18:03

Bonjour,
je passe par là mais je ne peux pas sortir directement un développement asymptotique!
Je peux donner une méthode qui permet de calculer les développements asymptotiques avec une précision aussi grande que l'on veut mais les calculs sont vite rébarbatifs.

1) Pour v_n la formule donnée par carpediem le 21-12-19 à 17:04 permet de le faire :

v_n = n + r_n avec  r_n = \dfrac {\ln n} n \left(1 +  \dfrac1 {\ln n} \ln (1 + \dfrac {r_n} n)\right) \left( 1 + \dfrac {r_n} n  \right)^{-1} (*)

En reportant r_n dans (*) on obtient d'abord :  r_n = \dfrac {\ln n} n \left(1 -  \dfrac {\ln n}{n^2}+ \dfrac1 {n^2}+o( \dfrac1 {n^2})\right)

Puis :

 r_n = \dfrac {\ln n} n \left(1 -  \dfrac {\ln n}{n^2}+ \dfrac1 {n^2}+\dfrac {2(\ln n)^2}{n^4}-\dfrac {7\ln n}{2n^4}+ \dfrac1 {n^4}+o( \dfrac1 {n^4})\right) (sauf erreur de calcul).

2) Pour u_n on peut utiliser, en posant u_n=\dfrac{w_n\ln n}{n} :

w_n=1-\dfrac{\ln(\ln n)}{\ln n}-\dfrac{\ln(w_n)}{\ln n}+\dfrac{w_n^2\ln n}{n^2} (**) . Le dernier terme étant négligeable devant tous les termes ayant une puissance de \ln n au dénominateur on peut le négliger.

En reportant w_n dans (**) on obtient : w_n=1-\dfrac{\ln(\ln n)}{\ln n}+\dfrac{\ln(\ln n)}{(\ln n)^2}+\dfrac{(\ln(\ln n))^2}{2(\ln n)^3}-\dfrac{\ln(\ln n)}{(\ln n)^3}+... (sauf erreur de calcul).

Posté par
lake
re : suite et limite 22-12-19 à 19:38

Merci jandri d'être « passé par ici. »

Posté par
carpediem
re : suite et limite 22-12-19 à 19:52

ha oui en reportant r_n dans r_n    on a plus mieux bien !!!

merci jandri ...

mais une question : pourquoi poses-tu pour w_n ... ce que tu poses ?

Posté par
jandri Correcteur
re : suite et limite 22-12-19 à 22:14

Pour un développement asymptotique de u_n j'ai cherché à écrire une formule permettant d'augmenter la précision à chaque étape.

J'ai d'abord recherché un équivalent de u_n en écrivant n\sim n-u_n= -\dfrac{\ln(u_n)}{u_n} (on voit tout de suite que u_n tend vers 0). En prenant le logarithme on obtient : \ln n\sim \ln(-\ln(u_n))-\ln(u_n)\sim -\ln(u_n) puisque \ln(-\ln(u_n)) est négligeable devant -\ln(u_n) qui tend vers +\infty.
On obtient -\ln(u_n)\sim\ln n d'où enfin u_n\sim\dfrac{\ln n}n.

Ensuite j'ai posé u_n=\dfrac{w_n\ln n}{n} (donc w_n tend vers 1). En remplaçant u_n dans n-u_n= -\dfrac{\ln(u_n)}{u_n} on obtient l'égalité (**). En négligeant le dernier terme on obtient une égalité qui se prête bien à l'augmentation de la précision à chaque étape.

Posté par
carpediem
re : suite et limite 23-12-19 à 09:39

ok merci ... je ne comprenais pas car ton w_n ne correspond pas à mon w_n : le produit  u_n * v_n ...

Posté par
jandri Correcteur
re : suite et limite 23-12-19 à 10:03

@carpediem
J'avais oublié que tu avais déjà utilisé w_n pour le produit u_nv_n.

Pour celui-ci seul le développement asymptotique de u_n intervient car on peut négliger tous les termes de v_n qui ont une puissance de n au dénominateur. On obtient :

 u_nv_n=\ln(n)-\ln(\ln n)}+\dfrac{\ln(\ln n)}{(\ln n)}+\dfrac{(\ln(\ln n))^2}{2(\ln n)^2}-\dfrac{\ln(\ln n)}{(\ln n)^2}+...

Posté par
LittleFox
re : suite et limite 23-12-19 à 11:35


Oh tiens, c'est bizarre, ça ressemble tout à fait au développement asymptotique de W_0 dont je parle depuis le début

En posant x = e^z on obtient facilement f(x) = 0 \Rightarrow z = e^z(e^z -n).

Or la fonction g(z) = e^z(e^z - n) est quasi verticale pour les z \approx ln(n). Ce qui donne x \approx e^{ln(n)} = n

De l'autre côté, quand z est négatif, e^z << n et donc g(z) \approx -ne^z.
Posant z = -ne^z on obtient -ze^{-z} = n et donc z = -W_0(n).
Et pour finir x \approx e^{-W_0(n)} = \frac{W_0(n)}{n}.

Donc u_n \approx  \frac{W_0(n)}{n}, v_n \approx n et u_nv_n \approx W_0(n).

Posté par
LittleFox
re : suite et limite 23-12-19 à 12:00


Et Wolfram Alpha donne une formule asymptotique pour n >= 3 avec aussi précise que l'on veut et plus facile à mettre en œuvre que celle de jandri mais qui donne bien les même résultats :

W_0(n) = ln(n) - ln(ln(z)) + \sum_{k=0}^{\infty}\sum_{m=0}^{k} \binom{k}{m} (ln(ln(n))^{m+1}(ln(n))^{-k-m-1}

Posté par
jandri Correcteur
re : suite et limite 23-12-19 à 22:03

@ LittleFox

c'est vrai que le développement asymptotique de nu_n est le même que celui de la fonction W_0 de Lambert mais celui-ci n'est pas aussi simple que tu l'écris : les coefficients qui interviennent ne sont pas du tout les coefficients binomiaux {k\choose m} et je ne crois pas qu'il y ait de formule simple pour les exprimer.

Posté par
jandri Correcteur
re : suite et limite 23-12-19 à 23:56

@ LittleFox

je rectifie ce que je viens d'écrire, il y a bien une formule.
Dans ta formule il faut remplacer le coefficient binomial {k\choose m} par (-1)^k\dfrac{|Stirling_1(1+m+k,1+k)|}{(m+1)!}Stirling_1 désigne le coefficient de Stirling de première espèce. Il faut également faire aller m de 0 à +\infty.

Posté par
jandri Correcteur
re : suite et limite 24-12-19 à 09:05

Encore une rectification, ce n'est pas une somme de série mais une somme finie.
La somme double porte sur tous les couples (k,m) d'entiers naturels vérifiant k+m\leq p pour un développement asymptotique avec une précision en o\left(\dfrac{(ln(ln(n)))^{p+1}}{(ln(n))^{p+1}}\right)



Vous devez être membre accéder à ce service...

Pas encore inscrit ?

1 compte par personne, multi-compte interdit !

Ou identifiez-vous :


Rester sur la page

Désolé, votre version d'Internet Explorer est plus que périmée ! Merci de le mettre à jour ou de télécharger Firefox ou Google Chrome pour utiliser le site. Votre ordinateur vous remerciera !